משתמש:אור שחף/133 - הרצאה/3.5.11

מתוך Math-Wiki
קפיצה אל: ניווט, חיפוש

את רשימת המשפטים לאינטגרלים לא אמיתיים מסוג II לא סיימנו בשיעור הקודם ולכן השלמנו זאת ב־3.5.11. חלק זה מופיע בסיכום השיעור הקודם ולא בדף הנוכחי.

אינטגרל לא אמיתי, סוג II (המשך)

דוגמה

\int\limits_0^{2\pi}\frac{\sin(x)}{\sqrt x\sqrt{|x-\pi|}^3}\mathrm dx - מתכנס או מתבדר?

נסמן f(x)=\frac{\sin(x)}{\sqrt x\sqrt{|x-\pi|}^3}. לפונקציה יש נקודת אי-רציפות סליקה באפס כי \lim_{x\to0^+}f(x)=\lim_{x\to0^+}\frac{\sin(x)}{x}\cdot\frac {x}{\sqrt x}\cdot\frac1{\sqrt{|x-\pi|}^3}=1\cdot0\cdot\frac1{\sqrt \pi^3}=0. כמו כן יש נקודת אי-רציפות ממין שני רק ב-\pi ולכן ונרשום: I_1:=\int\limits_0^\pi f\ \and\ I_2:=\int\limits_\pi^{2\pi} f.

f אי-שלילית בקטע [0,\pi]. לכן נגדיר g(x):=\frac1{\sqrt{x-\pi}} ונחשב \lim_{x\to\pi^-}\frac{f(x)}{g(x)}=\lim_{x\to\pi^-}
\frac{\sin(x)}{\sqrt x(\pi-x)}=\frac1{\sqrt\pi}\lim_{x\to\pi^-}\frac{\sin(x)}
{\pi-x}=\frac1{\sqrt\pi}\lim_{x\to\pi^-}\frac{\cos(x)}{-1}=\frac1{\sqrt\pi}\in\mathbb R ולכן I_1 מתכנס אם \int\limits_0^\pi g מתכנס, מה שאכן מתקיים: \int\limits_0^\pi g=\int\limits_0^\pi(\pi-x)^{-1/2}\mathrm dx=\left[-2\sqrt{\pi-x}\right]_{x=0}^\pi=2\sqrt\pi. באותו אופן אפשר להוכיח התכנסות I_2 (השוואה עם \frac{-1}{\sqrt{x-\pi}}). מכאן שאינטגרל הנתון מתכנס. \blacksquare



נושא שני:
סדרות וטורים של פונקציות

הגדרה: תהי \{f_n\}_{n=1}^\infty סדרת פונקציות המוגדרות כולן בקטע I. לכל x_0\in I נקבל סדרת מספרים \{f_n(x_0)\}_{n=1}^\infty ואפשר לדון ב-\lim_{n\to\infty} f_n(x_0). נגדיר את "תחום ההתכנסות" J\subseteq I של הסדרה כ-J:=\left\{x\in I:\lim_{n\to\infty}f_n(x)\in\mathbb R\right\}. כמו כן מוגדרת "פונקציה גבולית" f:J\to\mathbb R כך ש-f=\lim_{n\to\infty}f_n.

יש 2 נקודות מבט בהן ניתן להסתכל על סדרת פונקציות:

  1. סדרת פונקציות \{f_n\}_{n=1}^\infty היא פשוט אינסוף סדרות של מספרים \{f_n(x)\}_{n=1}^\infty, עם x\in I לכל סדרה. זהו מבט נקודתי.
  2. סדרת פונקציות היא, כשמה, סדרה של פונקציות ששואפות לפונקציה חדשה - הפונקציה הגבולית. זהו מבט פונקציונלי.

הגדרה: נניח שיש לנו סדרת פונקציות \{u_n\}_{n=1}^\infty על I. אפשר לבנות טור \sum_{n=1}^\infty u_n(x) כאשר התכנסות הטור נקבעת עפ"י הסכומים החלקיים S_N(x)=\sum_{n=1}^N u_n(x) וה-\{S_N\}_{N=1}^\infty סדרת פונקציות על I. תחום ההתכנסות ל-S_N, לפי ההגדרה, J=\left\{x\in I:\lim_{N\to\infty}S_N(x)=\sum_{n=1}^\infty u_n(x)\in\mathbb R\right\}. כמו כן הפונקציה הגבולית של הסדרה היא S(x)=\lim_{N\to\infty}S_N(x).

דוגמאות

  1. \forall n\in\mathbb N:\ f_n(x)=x^n. זאת סדרת פונקציות על \mathbb R ומתקיים f(x)=\lim_{n\to\infty}x^n=\begin{cases}0&|x|<1\\1&x=1\\\text{undefined}&\text{else}\end{cases}. לפיכך תחום ההתכנסות הוא הקטע J=(-1,1]. נשים לב כי יש לפונקציה הגבולית נקודת אי-רציפות ב-x=1 אעפ"י שכל ה-f_n רציפות בנקודה זו.
  2. נחשב את הפונקציה הגבולית עבור f_n(x)=\frac{n^2x}{1+(nx)^2}. עבור x=0 מתקיים \forall n:\ f_n(0)=0. עבור x\ne0 נקבל \lim_{n\to\infty}\frac{n^2x}{1+(nx)^2}=\lim\frac x{\frac1{n^2}+x^2}=\frac1x. לכן הפונקציה הגבולית היא f(x)=\begin{cases}0&x=0\\\frac1x&x\ne0\end{cases}.
  3. הטור ההנדסי \sum_{n=0}^\infty x^n שווה ל-\begin{cases}\frac1{1-x}&|x|<1\\\text{undefined}&\text{else}\end{cases} (לפי נוסחת הסכום של טורים הנדסיים). תחום ההתכנסות הוא (-1,1).
  4. נבדוק למה שווה הטור \sum_{n=1}^\infty nx^n עבור |x|<1:
    \begin{align}\sum_{n=1}^\infty nx^n&=\left(x+x^2+x^3+\dots\right)+\left(x^2+x^3+\dots\right)+\left(x^3+\dots\right)+\dots\\&=\frac x{1-x}+\frac{x^2}{1-x}+\frac{x^3}{1-x}+\dots\\&=\frac x{1-x}\sum_{n=0}^\infty x^n\\&=\frac x{(1-x)^2}\end{align}
    \blacksquare
    גישה אחרת (מבט פונקציונלי): נגדיר S(x)=\sum_{n=1}^\infty x^n=\frac1{1-x}. אם יש צדק בעולם S'(x)=\sum_{n=1}^\infty nx^{n-1} ולכן \sum_{n=1}^\infty nx^n=x\cdot S'(x)=x\cdot\frac1{(1-x)^2}, אלא שאנו זקוקים למשפט כדי להצדיק את גזירת הטור איבר-איבר אינסוף פעמים (כלומר משפט האומר ש-\left(\sum_{n=1}^\infty f_n(x)\right)'=\sum_{n=1}^\infty f_n'(x)), ולא הוכחנו כזה דבר (אך נעיר שעבור הטור הזה זה נכון).
  5. נגדיר f_n(x)=\frac{\sin\left(n^2x\right)}n. לכן הפונקציה הגבולית היא f(x)=\lim_{n\to\infty}\sin\left(n^2x\right)\frac1n=0. אם יש צדק בעולם אז f_n(x)\to0\implies f_n'(x)\to0'=0, אלא שצדק נמצא בחלל ובפרט f_n'(x)=n\cos\left(n^2x\right) ולכן \lim_{n\to\infty}f_n'(x) לא קיים לאף x\in\mathbb R.
  6. נתבונן בטור S(x)=\sum_{n=0}^\infty\frac{x^n}{n!} ונוכיח כי \forall x\in\mathbb R:\ S(x)=e^x. נעשה זאת באמצעות טורי טיילור: e^x=P_N(x)+R_N(x) וכבר הראנו בקורס אינפי 1 ש-\forall x\in\mathbb R:\ P_N(x)=\sum_{n=0}^N\frac{x^n}{n!}\ \and\ R_N(x)=\frac{f^{(N+1)}(c)}{(N+1)!}x^{N+1}=\frac{e^c}{(N+1)!}x^{N+1} עבור c כלשהו בין 0 ל-x. כעת הטור הנתון מקיים S(x)=\lim_{N\to\infty}P_N(x)=\lim_{N\to\infty} e^x-R_N(x). כדי להראות ש-S(x)=e^x נותר להוכיח ש-\lim_{N\to\infty}R_N(x)=0. ובכן נקח x\in\mathbb R כרצונינו ונשים לב כי לכל N כך ש-x<N\in\mathbb N מתקיים
    0\le|R_N(x)|\le\frac{e^{|x|}|x|^{N+1}}{(N+1)!}=e^{|x|}\frac{|x|}1\frac{|x|}2\frac{|x|}3\cdots\frac{|x|}{\lfloor x\rfloor}\frac{|x|}{\lfloor x\rfloor+1}\cdots\frac{|x|}{\lfloor x\rfloor+(N+1-\lfloor x\rfloor)}\to0
    וכך הוכחנו בעזרת המבט הנקודתי. \blacksquare עתה ננסה להוכיח גם מנקודת מבט פונקציונלית: S(x)=1+x+\frac{x^2}{2!}+\frac{x^3}{3!}+\frac{x^4}{4!}+\dots ולכן S'(x)=0+1+x+\frac{x^2}{2!}+\frac{x^3}{3!}+\dots=S(x). נגדיר f(x)=S(x)\cdot e^{-x} ולכן f'(x)=S'(x)e^{-x}+S(x)\left(-e^{-x}\right)=S(x)e^{-x}-S(x)e^{-x}=0 ומכאן ש-f פונקציה קבועה. נסמן c=f(x) ונובע ש-S(x)=f(x)e^x=ce^x. מהגדרת S נובע כי S(0)=1 ז"א 1=S(0)=ce^0=c, ולכן S(x)=e^x ובפרט e=e^1=\sum_{n=0}^\infty\frac1{n!}. ו"הוכחנו" את הטענה (לצערנו גזרנו טור אינסופי איבר-איבר, אבל כאמור, אין לנו משפט שאומר שזה נכון).




טענה: e אינו רציונלי.

הוכחה: נניח בשלילה ש-e רצינלי ונסמן e=\frac pq עבור p,q\in\mathbb N. לכן q!e=(q-1)!p\in\mathbb N, אבל q!e=\underbrace{q!+q!+\frac{q!}{2!}+\frac{q!}{3!}+\dots+\frac{q!}{q!}}_{\in\mathbb N}+\underbrace{\frac1{q+1}+\frac1{(q+1)(q+2)}+\dots}_{<\sum_{n=1}^\infty\frac1{q^n}=\frac1{q-1}<1}, כלומר q!e הוא מספר טבעי השווה למספר טבעי ועוד מספר לא שלם, בסתירה. \blacksquare

התכנסות במידה שווה

הגדרה: תהי \{f_n\} סדרת פונקציות בקטע I כך שלכל x\in I קיים הגבול f(x)=\lim_{n\to\infty} f_n(x). ניתן שתי הגדרות שקולות לשאיפה של f_n ל-f במידה שווה ב-I:

  • לכל \varepsilon>0 קיים n_0\in\mathbb N כך שאם n>n_0 אז |f(x)-f_n(x)|<\varepsilon לכל x\in I.
  • \lim_{n\to\infty}\sup_{x\in I} |f(x)-f_n(x)|=0.